--- /dev/null
+\begin{problem*}{26.61}
+Calculate the three currents $I_1$, $I_2$, and $I_3$ indicated in the
+circuit diagram shown in Fig.~26.65.
+\begin{center}
+\begin{verbatim}
+ 5.00 8.00
++------/\/\/------+------/\/\/------+
+|12.00V 1.00 I2 | I1 1.00 9.00V |
++--|i---/\/\/--<--+-->--/\/\/---i|--+
+| I3 10.00 |
++-------->------/\/\/---------------+
+\end{verbatim}
+\end{center}
+\end{problem*}
+
+\begin{solution}
+\end{solution}
--- /dev/null
+\begin{problem*}{26.86}
+An $R$-$C$ circuit has a time constant $RC$. \Part{a} If the circuit
+is discharging, how long will it take for its stored energy to be
+reduced to $1/e$ of its initial value? \Part{b} If it is charging,
+how long will it take for the stored energy to reach $1/e$ of its
+maximum value?
+\end{problem*}
+
+\begin{solution}
+\end{solution}
--- /dev/null
+\begin{problem*}{26.91}
+As shown in Fig.~26.83, a network of resistors of resistances $R_1$
+and $R_2$ extends to infinity toward the right. Prove that the total
+resistance $R_T$ of the infinite network is equal to
+\begin{equation}
+ R_T = R_1 + \sqrt{R_1^2 + 2R_1R_2}
+\end{equation}
+(\emph{Hint:} Since the network is infinite, the sestance of the
+network to the right of points $c$ and $d$ is also equal to $R_T$.)
+\begin{center}
+\begin{verbatim}
+ R1 R1 R1
+a-/\/\/-c-/\/\/-+-/\/\/-+-...
+ | | |
+ Z Z Z
+ Z R2 Z R2 Z R2
+ Z Z Z
+ R1 | R1 | R1 |
+b-/\/\/-d-/\/\/-+-/\/\/-+-...
+\end{verbatim}
+\end{center}
+\end{problem*}
+
+\begin{solution}
+\end{solution}
--- /dev/null
+\begin{problem*}{27.22}
+In an experiment with cosmic rays, a verticle beam of particles that
+have chagre of magnitude $3e$ and mass $12$ times the proton mass
+enters a uniform horizontal magnetic field of $0.250\U{T}$ and is bent
+in a semicircle of diameter $95.0\U{cm}$, as shown in
+Fig.~27.47. \Part{a} Find the speed of the particles and the sign of
+their charge. \Part{b} Is it reasonable to ignore the gravity force
+on the particles? \Part{c} How does the speed of the particles as
+they enter the field compare to their speed as they exit the field?
+\end{problem*}
+
+\begin{solution}
+\end{solution}
--- /dev/null
+\begin{problem*}{27.30}
+A particle with initial velocity $\vect{v}_0=5.85\E{3}\U{m/s}\jhat$
+enters a region of uniform electric and magnetic fields. The magnetic
+field in the region is $\vect{B}=-(1.35\U{T})\khat$. Calculate the
+magnitude and direction of the electric field in the region if the
+particle is to pass through undeflected, for a particle of
+charge \Part{a} $+0.640\U{nC}$ and \Part{b} $-0.640\U{nC}$. You can
+ignore the weight of the particle.
+\end{problem*}
+
+\begin{solution}
+\end{solution}
--- /dev/null
+\begin{problem*}{27.35}
+A long wire carrying $4.50\U{A}$ of current makes two $90\dg$ bends,
+as shown in Fig.~27.49. The bent part of the wire passes through a
+uniform $0.240\U{T}$ magnetic field direceted as shown in the figure
+and confined to a limited region of space. Find the magnitude and
+direction of the force that the magnetic field exerts on the wire.
+\begin{center}
+\begin{asy}
+import ElectroMag;
+
+real u = 2.5cm;
+
+Distance Dhorizontal = Distance((0,0),(u,0), offset=2mm, L="$60.0\U{cm}$");
+Distance Dvertical = Distance((u,0),(u,u), offset=2mm, L="$60.0\U{cm}$");
+Distance Dbend = Distance((.25u,.25u),(.25u,.75u), offset=2mm, L="$30.0\U{cm}$");
+
+draw(scale(u)*((0,0)--(1,0)--(1,1)--(0,1)--cycle), blue);
+draw(scale(u)*((-.25,.25)--(.25,.25)--(.25,.75)--(1.25,.75)), red);
+Dhorizontal.draw(labelangle=-90, labeloffset=8pt);
+Dvertical.draw(labelangle=-90, labeloffset=8pt);
+Dbend.draw(rotateLabel=false, labelangle=-90, labeloffset=22pt);
+\end{asy}
+\end{center}
+\end{problem*}
+
+\begin{solution}
+\end{solution}